Calculate the size of angle 0.
Give your answer to the nearest degree.

Calculate The Size Of Angle 0.Give Your Answer To The Nearest Degree.

Answers

Answer 1

In the given triangle, using the law of Cosines, the value of angle θ is 103°

Law of Cosines: Calculating the value of an angle

From the question, we are to calculate the value of angle θ in the diagram.

From the given diagram,

We have a triangle ABC with given side lengths

From the given information,

a = 42 cm

b = 78 cm

c = 57 cm

From the law of Cosines, we have that

cos B = (a² + c² - b²)/2ac

Thus,

cos θ = (a² + c² - b²)/2ac

Substitute the parameters

cos θ = (42² + 57² - 78²)/2(42)(57)

cos θ = (1764 + 3249 - 6084) / (4788)

cos θ = -1071/4788

θ = cos⁻¹(-1071/4788)

θ = 102.9255

θ ≈ 103°

Hence, the value of θ is 103°

Learn more on Law of Cosines here: https://brainly.com/question/28022160

#SPJ1


Related Questions

An airplane flight has 228 seats. The probability that a person who buys a ticket actually goes on that flight is about 95%. If the airline wants to fill all the seats on the flight, how many tickets should it sell?
345 tickets
240 tickets
2400 tickets
217 tickets

Answers

Answer:

240 tickets sold

Step-by-step explanation:

first, you would set up the proportion by writing [tex]\frac{228}{95} =\frac{x}{100}[/tex]

with "x" being the number of seats that needs to be sold

then, you would cross multiply

and you get

[tex]95x=228*100[/tex]    next, you simplify to get the inequality [tex]95x=22800[/tex]

then divide both sides by 95

[tex]x=240[/tex]

so 240 tickets to fill all the seats

hope this helps ;)

Altogether the Green Team jumped 10264 times. The Yellow Team did 759 fewer jumps than the Green Team. How many jumps did the Yellow Team do?

Answers

So fewer stand for subtract between the 2 teams. So we will be doing 10264-759 which that would be 9505 that the yellow team did.

A rectangular hotel room is 3 meters by 8 meters. The owner of the hotel wants to recarpet the room with carpet that costs $33.38 per square meter. How much will it cost to recarpet the room?

Answers

The amount it will cost to recarpet the the whole room is $891.12

What is area ?

The area of a shape is the space occupied by the boundary of a plane figures like circles, rectangles, and triangles.

For example, the area of a circle is given as: A = πr², where r is the radius and π is constant.

The room takes the shape of a rectangle, therefore,

Area of the room = l× w, where l is the length and w is the width.

Area = 3×8 = 24 m²

The cost of 1m² = $33.38

therefore the cost for 24m²

= 24× 33.8

= $801.12

therefore the cost to recarpet the room is $801.12

learn more about area from

https://brainly.com/question/25292087

#SPJ1

2. A pair of jeans is on sale for 25% off the original price. Which expression represents the sale
price? If the original price of the jeans is $40, evaluate the expression to find the sale price.
Os=p-0.25p; $30
Os=p-25; $15
Os=p-0.25p; $10
Os p-0.25; $39.75

Answers

Answer: s = p - 0.25p; $30

Step-by-step explanation:

      First, we need to know that a percent divided by 100 becomes a decimal.

          25% / 100 = 0.25.

      Next, we will use this equation because it takes 25% off of the original price (p) by subtracting 0.25p from p.

          s = p - 0.25p

      Lastly, we will substitute 40 in for 0 and solve.

          s = ($40) - 0.25($40)

          s = $30

what is the value of x, in units?

Answers

Answer: 12 units

Step-by-step explanation: see both shapes are the same just one is flipped and its small meaning the first shape without numbers is only double of the shape with numbers so you answer would be 12 units

small shape x 2 gives all your answers to the big shape

TOO

EASY

BYE

Write a quadratic function f whose zeros are -2 and 12.

f(x) = 0

Answers

Answer:

[tex]f(x) = (x + 2)(x - 12)[/tex]

[tex]f(x) = {x}^{2} - 10x - 24[/tex]

Question 5
Karin has $83,555 in a retirement account right now. Suppose she deposits $3,000 this year and then increases that by 7.5% each subsequent year. The account earns 9.8%. Find her future account value after 20 years.

Answers

The solution of the given problem of percentage comes out to be Karin's future account value would therefore be about $313,425.67 after 20 years.

What are percentages?

In statistics, a figure or metric that may be presented as a percentage or 100 is denoted by the abbreviation "a%". Other unusual spelling is "pct," "pct," and "pc." The percent symbol ("%") is the method that is most usually used for this.

Here,

Using the compound interest calculation, we can determine Karin's account worth in 20 years:

=> A = P(1 + r)ⁿ

P =$83,555 (current balance), as given.

(Annual interest rate) r = 9.8% = 0.098

20 years, n

Let's use the following formula to determine the future account worth for each year:

Year 1: P = $83,555 (balance as of now)

(Annual interest rate) r = 9.8% = 0.098

Given that this is the first year, n equals 1.

=> A = $83,555(1 + 0.098)¹

=> A = $83,555(1.098)

=>A = $91,785.09

Year 2: P = $6,225 (current balance after deposit) = $3,000 + $3,225.

(Annual interest rate) r = 9.8% = 0.098

n = 1 year

=> A = $6,225(1 + 0.098)¹

=> A = $6,225(1.098)

=> A = $6,834.44

After 20 years, the current balance is

=>  P = $12,873.51 + (20 - 1)($3,000 + 7.5% of $3,000)

= $12,873.51 + $71,132.10 = $83,005.61.

The yearly interest rate is 9.8%, or 0.098.

20 years, n

=>  A = $83,005.61(1 + 0.098)²⁰

=>   A = $83,005.61(3.778)

=>  A = $313,425.67

Karin's future account value would therefore be about $313,425.67 after 20 years.

To know more about percentage visits:

https://brainly.com/question/31471114

#SPJ1

The power of -1 to a function, or ((f^-1)(x)), means...

Answers

The power of -1 to a function, or [tex]((f^{-1})(x)),[/tex]represents the inverse function of f. In other words, if f(x) produces a certain output y, then the inverse function[tex]f^{-1}(y)[/tex]will produce the input x. It is important to note that not all functions have an inverse, and if they do, they may only be valid for certain inputs and outputs.

In mathematics, the inverse function of a function f (also called the inverse of f) is a function that undoes the operation of f1. The inverse of f exists if and only if f is bijective.

[tex]((f^{-1})(x)),[/tex]

The inverse function of f is denoted by. For a function, its inverse admits an explicit description: it sends each element to the unique element such that f(x) = y1.

learn more about inverse function

https://brainly.com/question/2541698

#SPJ11

7x-6° 120° Find the value of x

Answers

Answer:

x = 18°

Step-by-step explanation:

7x - 6° = 120°

7x = 126°

x = 18°

So, the answer is x = 18°

what is the maximum number of mail swaps bagelbot can perform between officials from a and officials from b? assume every official regularly receives a lot of mail, and bagelbot can redirect any of it anywhere at any time. (i.e., what is the maximum number of edges possible for a bipartite graph between a and b?)

Answers

The maximum number of mail swaps that Bagelbot can perform between officials from A and officials from B can be 150 .

It can be determined by calculating the maximum number of edges possible for a bipartite graph between A and B. A bipartite graph is a graph in which the vertices can be divided into two disjoint sets such that every edge connects a vertex in one set to a vertex in the other set.

In this case, the officials from A and officials from B represent the two disjoint sets.

The maximum number of edges in a bipartite graph between A and B can be calculated as the product of the number of officials in A and the number of officials in B. Therefore, if there are n officials in A and m officials in B, the maximum number of mail swaps that Bagelbot can perform is n x m.

For example, if there are 10 officials in A and 15 officials in B, the maximum number of mail swaps that Bagelbot can perform is 10 x 15 = 150.

This means that Bagelbot can potentially redirect up to 150 pieces of mail between officials from A and officials from B.

Know more about   maximum number   here:

https://brainly.com/question/29795588

#SPJ11

Question 8(Multiple Choice Worth 2 points)
(Creating Graphical Representations MC)

The number of milligrams of Vitamin C from 100 different gummy vitamins sold in the world was collected.

Which graphical representation would be most appropriate for the data, and why?

Box plot, because the median can easily be determined from the large set of data
Stem-and-leaf plot, because you can see the shape of the data
Histogram, because it shows each individual data point
Bar chart, because the data is categorical

Answers

The box plot, as the median can be easily estimated from the big set of data, is the graphical representation that's most suitable for the data for gummy vitamins sold globally.

Explain about the box plot:

The distribution of data is shown using a boxplot, which is a standardised method based on a five-number summary ("minimum," first quartile ("Q1"), median ("Q3"), and "maximum").

It can reveal information about your outliers' values. Boxplots can also show you how tightly that data is grouped, whether or not your data is skewed, and whether or not your data is symmetrical.The lower and upper quartiles are shown on the left and right sides of a box and whisker plot, respectively. The interquartile range, which contains 50% of the data, is covered by the box. The median is the vertical line which it divided the box in half.

For the case of -

A total of 100 distinct gummy vitamins that are offered worldwide were counted for their milligrammes of vitamin C content.

The box plot, as the median can be easily estimated from the big set of data, is the graphical representation that's most suitable for the data for gummy vitamins sold globally.

Know more about the box plot:

https://brainly.com/question/28992381

#SPJ1

$480, 15% interest, 1.5 years. Find the interest to the nearest cent for each principal, interest rate, and time.

Answers

The simple interest earned after 3 years on $500 at an interest rate of 6% is $90.

Simple interest is a type of interest that is calculated based only on the initial amount borrowed or invested, called the principal amount, and does not take into account any additional interest earned on the interest over time.

To find the simple interest earned on $500 at an interest rate of 6% after 3 years, we can use the formula:

Simple Interest = Principal x Rate x Time

Where

Principal = $500

Rate = 6% = 0.06 (in decimal form)

Time = 3 years

Plugging these values into the formula, we get

Simple Interest = $500 x 0.06 x 3

= $90

Learn more about simple interest here

brainly.com/question/25845758

#SPJ4

I have solved the question in general, as the given question is incomplete.

The complete question is:

Find the simple interest earned after 3 years on $500 at an interest rate of 6%.

The center of a circle is at (−2, −7) and its radius is 6.

What is the equation of the circle?

Responses

(x+2)2+(y+7)2=3
open parenthesis x plus 2 close parenthesis squared plus open parenthesis y plus 7 close parenthesis squared equals 3

(x+2)2+(y+7)2=36
open parenthesis x plus 2 close parenthesis squared plus open parenthesis y plus 7 close parenthesis squared equals 36

(x−2)2+(y−7)2=3
open parenthesis x minus 2 close parenthesis squared plus open parenthesis y minus 7 close parenthesis squared equals 3

(x−2)2+(y−7)2=36

Answers

option B is correct: [tex](x+2)^2+(y+7)^2=36[/tex]

The general equation of the circle is given by:

[tex](x-h)^2+(y-k)^2=r^2[/tex]              

where,

(h, k) is the center of the circle and r is the radius of the circle.

As per the statement:

The center of a circle is at (−2, −7) and its radius is 6.

[tex]\implies (h, k) = (-2, -7)[/tex] and [tex]r = 6[/tex] units

Substitute these we have:

[tex](x-(-2))^2+(y-(-7))^2=6^2[/tex]

[tex]\implies(x+2)^2+(y+7)^2=36[/tex]

Therefore, the equation of circle is, [tex]\bold{(x+2)^2+(y+7)^2=36}[/tex]

Javae got $100 for his birthday. He listened to a Podcast about investing, and decided he would try it. He invested his $100 into a fund that has 9% growth per year. How much will Javae have in that account on his 30th birthday? (16 years
from now).

O 1,327
O $22
O $2,884,414
O $397

Answers

Answer:

The formula for compound interest is A = P(1 + r/n)^(nt), where A is the final amount, P is the initial principal, r is the interest rate, n is the number of times the interest is compounded per year, and t is the time in years.

In this case, P = $100, r = 9% = 0.09, n = 1 (compounded annually), and t = 16. We want to find A.

A = 100(1 + 0.09/1)^(1*16)

A = $397.42

Therefore, Javae will have $397.42 in his account on his 30th birthday. The answer is option D.

QUICK QUICKQUICK QUICKQUICK QUICKQUICK QUICKQUICK QUICKQUICK QUICKQUICK QUICKQUICK QUICKQUICK QUICKQUICK QUICKQUICK QUICKQUICK QUICKQUICK QUICKQUICK QUICK

Answers

Answer:

  15.0 in

Step-by-step explanation:

You want the height of a cone with radius 8 in and slant height 17 in.

Pythagorean theorem

The right triangle shown in the figure has height h, base x = 8 in, and slant height y = 17 in. The Pythagorean theorem tells you the relationship between these side lengths:

  x² +h² = y²

  8² +h² = 17²

  h² = 17² -8² = 289 -64 = 225

  h = √225 = 15.0

The height of the cone is 15.0 inches.

__

Additional comment

A triple of 3 integers that form the sides of a right triangle is called a "Pythagorean triple." The triple in this problem is one of those: {8, 15, 17}. Other Pythagorean triples you will often see in algebra, trig, and geometry problems are {3, 4, 5}, {5, 12, 13}, {7, 24, 25}, {9, 40, 41}. You will also see multiples of these, for example 3×{3, 4, 5} = {9, 12, 15}.

It can be worthwhile to remember some of these. For this problem, recognizing 8 and 17 as part of the triple {8, 15, 17} means you can write down the answer with no further work.

the graph below displays the amount of time to the nearest hour spent on homework per week for a sample of students. which measures of center and variability would be most appropriate to describe the given distribution? group of answer choices mean and standard deviation mean and iqr median and standard deviation median and iqr median and range

Answers

The Median and IQR are most appropriate to describe the measures of center and variability of distribution. The histogram depicts the distribution of time is positively skewed. So, option(b) is right one.

We have a graph present in above figure, which showes the amount of time in hours spent on homework per week for a sample of students. We have to select correct measures of center and variability. The graph is a histogram. From the graph we can see that the data is positively skewed. Now for skewed distribution we prefer median over mean as mean is more sensitive to the outliers. As the data(time) is in ratio scale we can use any of the measures of dispersion; but for skewed distribution IQR is preferred because of its robustness. In a word both median and IQR can handle outliers. Thus the appropriate answer is Median and IQR, option (c).

For more information about Median, visit:

https://brainly.com/question/26177250

#SPJ4

Complete question :

The above figure complete the question

the graph below displays the amount of time to the nearest hour spent on homework per week for a sample of students. which measures of center and variability would be most appropriate to describe the given distribution? group of answer choices

a) mean and standard deviation

b) mean and iqr

c) median and standard deviation

d) median and iqr

e) median and range

Depreciation by Three Methods; Partial Years Perdue Company purchased equipment on April 1 for $41,550. The equipment was expected to have a useful life of three years, or 5,700 operating hours, and a residual value of $1,650. The equipment was used for 1,000 hours during Year 1, 2,000 hours in Year 2, 1,700 hours in Year 3, and 1,000 hours in Year 4. Required: Determine the amount of depreciation expense for the years ended December 31, Year 1, Year 2, Year 3, and Year 4, by (a) the straight-line method, (b) the units-of-activity method, and (c) the double-declining-balance method. Note: Round all final values for each depreciation method and each year to the nearest whole dollar

Answers

a) The depreciation expenses for Year 1 to Year 4 using the straight-line method are $2,632, $5,263, $4,211, and $2,632, respectively.

b) Using the units-of-activity method, the depreciation expenses are $7,000, $14,000, $11,900, and $7,000, respectively.

c) Using the double-declining-balance method, the depreciation expenses are $4,658, $11,172, $7,302, and $2,640, respectively.

a) To calculate the depreciation expense for each year using the straight-line method, the company needs to first determine the depreciable cost of the equipment. This is the cost of the equipment minus its estimated residual value.

The depreciable cost is $41,550 - $1,650 = $39,900.

To calculate the annual depreciation expense using the straight-line method, divide the depreciable cost by the useful life of the equipment, which is three years.

Year 1: ($39,900 ÷ 3) × (1,000 ÷ 5,700) = $2,632

Year 2: ($39,900 ÷ 3) × (2,000 ÷ 5,700) = $5,263

Year 3: ($39,900 ÷ 3) × (1,700 ÷ 5,700) = $4,211

Year 4: ($39,900 ÷ 3) × (1,000 ÷ 5,700) = $2,632

b) To calculate the depreciation expense using the units-of-activity method, the company needs to first determine the depreciation rate per unit of activity. This is the depreciable cost of the equipment divided by its total estimated hours of use.

The depreciation rate per unit of activity is $39,900 ÷ 5,700 hours = $7 per hour.

Year 1: $7 × 1,000 hours = $7,000

Year 2: $7 × 2,000 hours = $14,000

Year 3: $7 × 1,700 hours = $11,900

Year 4: $7 × 1,000 hours = $7,000

c) To calculate the depreciation expense using the double-declining-balance method, the company needs to first determine the straight-line depreciation rate. This is the depreciable cost of the equipment divided by its useful life.

The straight-line depreciation rate is $39,900 ÷ 3 years = $13,300 per year.

The double-declining-balance depreciation rate is twice the straight-line rate, or $13,300 × 2 = $26,600.

Year 1: $26,600 × (1,000 ÷ 5,700) = $4,658

Year 2: ($39,900 - $4,658) × (2,000 ÷ 5,700) = $11,172

Year 3: ($39,900 - $4,658 - $11,172) × (1,700 ÷ 5,700) = $7,302

Year 4: ($39,900 - $4,658 - $11,172 - $7,302) × (1,000 ÷ 5,700) = $2,640

To learn more about costs click on,

https://brainly.com/question/15224550

#SPJ4

If a reduced echelon matrix T(x) = 0 has a row of [ 0 . . 0 | 0] or [0 . . .0 | b] , where b =/= 0, it's considered one to one.
a. true
b. false

Answers

The correct answer is false. A reduced echelon matrix [tex]T(x) = 0[/tex] with a row of [tex][0 . . 0 | 0] or [0 . . . 0 | b][/tex], where[tex]b \neq 0[/tex], is not considered one-to-one.

Let's first define what it means for a matrix to be one-to-one.

A matrix is said to be one-to-one if each column of the matrix is linearly independent.

This means that for any given input, there is only one possible output.
Now, let's consider the reduced echelon matrix T(x) = 0. If this matrix has a row of [tex][0 . . 0 | 0] or [0 . . . 0 | b][/tex], b ≠ 0, it means that one of the variables in the system of equations represented by the matrix is a free variable.

This free v[tex]T(x) = 0.[/tex]ariable can take on any value, and the other variables will adjust accordingly.
If the reduced echelon matrix is [tex][1 0 | 2; 0 0 | 0],[/tex] the system of equations would be [tex]x = 2[/tex] and y can be any value.

This means that the matrix is not one-to-one, because there are multiple outputs for the same input.
the correct answer is false.

A reduced echelon matrix[tex]T(x) = 0[/tex] with a row of [tex][0 . . 0 | 0] or [0 . . . 0 | b][/tex], where [tex]b \neq 0[/tex], is not considered one-to-one.

For similar questions on matrix

https://brainly.com/question/1279486

#SPJ11

I need help solving this equation. I keep getting the +/- mixed up.
12-tx2-pn=12-t(2-x)2-pm
12-tx2-pn=12-4t+4tx-tx2-pm
Eliminate the 12 and tx2
-pn=-4t+4tx-pm
-4tx=-4t + pn-pm
divide both sides by -4t
x=1 - (pm-pn)/4t
I am not sure if this is right, but I have x= 1-(pm-pn)/4t

Answers

It looks like you did a great job solving the equation, and your final answer is correct. Here is a brief summary of the steps you took:
your final answer is correct[tex]: x = 1 - (pm - pn) / 4t[/tex]

Mathematics has been classified into many branches. Arithmetic is the branch that handles numbers and their operations. Arithmetic taught us addition, subtraction, multiplication and divisions of two or more numbers. Geometry is all about shapes and their construction using different tools like a compass, ruler and pencil. Algebra is another interesting branch where we express our daily life situations in numbers and letters (variables).
1. Expanded the equation: [tex]12 - tx^2 - pn = 12 - t(2 - x)^2 - pm[/tex]
2. Simplified: [tex]12 - tx^2 - pn = 12 - 4t + 4tx - tx^2 - pm[/tex]
3. Eliminated the 12 and tx^2 terms: -pn = -4t + 4tx - pm
4. Rearranged the equation:[tex]-4tx = -4t + pn - pm[/tex]
5. Divided both sides by [tex]-4t: x = 1 - (pm - pn) / 4t[/tex]
So, your final answer is correct[tex]: x = 1 - (pm - pn) / 4t[/tex]

learn more about Arithmetic

https://brainly.com/question/11559160

#SPJ11

Ok so I cant figure out how to find the angle of measure "C" I got measure "A" and "B" but don't understand "C"

Answers

Answer: The answer is 84

The total of the angels of the triangle always equals 180. So 62+34=96 therefor 96-180=84.


Help me on the questions please and thank you.

Answers

The lateral surface area of the rectangular prism is 308 units² and the total surface area is 398 units²

What is the lateral and total surface area of the rectangular prism

The formula of lateral surface area of the rectangular prism is given as;

LSA = 2(l +b)h

l = length b = breath or widthh = height

substituting the values into the formula;

LSA = 2(5 + 9) * 11

LSA = 308 units²

The total surface area is given by

TSA = 2(lb + bh + lh)

TSA = 2[(5*9) + (9*11) + (11*5)

TSA = 398 units²

Learn more on lateral surface area here;

https://brainly.com/question/15777841

#SPJ1

Divide. Give the Quotient and Remainder. 520 / 22

Answers

Answer:

Quotient=23 and remainder=14

the probability a visitor to the home page of a website views another page on the site is 0.2. assume that 200

Answers

The probability that at least one visitor views another page on the site is very high, close to 1. This suggests that the website is engaging and visitors are likely to navigate beyond the home page.

Assuming that 200 visitors come to the home page of the website, we can use the binomial distribution to calculate the probability that at least one visitor views another page on the site.

Let X be the number of visitors who view another page. Then X follows a binomial distribution with n = 200 and p = 0.2.

P(X ≥ 1) = 1 - P(X = 0)

[tex]= 1 - (0.2)^0 * (0.8)^200= 1 - 0.8^200[/tex]

≈ 1

Therefore, the probability that at least one visitor views another page on the site is very high, close to 1. This suggests that the website is engaging and visitors are likely to navigate beyond the home page.

Learn more about The probability

https://brainly.com/question/30034780

#SPJ4

40 out of 200 visitors to the homepage are expected to view another page on the site.

The probability that a visitor to a website's homepage views another page on the site and how it relates to 200

visitors.

The probability is 0.2.

To find out how many visitors out of 200 are likely to view another page on the site, you can follow these steps:

Identify the given probability, which is 0.2 in this case.

Multiply the probability by the total number of visitors (200).

So, the calculation would be:

0.2 × 200 = 40

Based on the given probability, approximately 40 out of 200 visitors to the homepage are expected to view another

page on the site.

for such more question on probability

https://brainly.com/question/13604758

#SPJ11

William is creating a film for a school project using a digital video camera.

William transfers the videos to a computer for editing.

(i) The computer has 1GB of storage free.

Calculate the number of videos that could be stored on the computer if each video was 100MB in size.

Show your working.

(ii) A program needs to calculate the size of files in bytes. The program must:

· Ask the user to input a file size in megabytes
· calculate and output the number of bytes this represents in a user friendly format.
(e.g. "There are 5242880 bytes in 5MB").

Write an algorithm using pseudocode to calculate the number of bytes in a given number of megabytes.

Answers

(i) To calculate the number of videos that could be stored on the computer:

1GB = 1000MB (since 1GB = 1024MB, but we'll use the simpler 1000MB approximation here)

Number of videos = (1GB free space) / (100MB per video)
Number of videos = (1000MB) / (100MB per video)
Number of videos = 10 videos

Therefore, if each video is 100MB in size, William can store a maximum of 10 videos on the computer.

(ii) Algorithm to calculate the number of bytes in a given number of megabytes:

1. Ask the user to input the file size in megabytes.
2. Read the input and store it in a variable, let's call it "megabytes".
3. Calculate the number of bytes by multiplying "megabytes" by 1,048,576 (this is the number of bytes in a megabyte).
4. Output the result in a user-friendly format, such as: "There are [number of bytes] bytes in [megabytes] MB."

Here's the pseudocode:

```
INPUT megabytes
bytes = megabytes * 1048576
OUTPUT "There are " + bytes + " bytes in " + megabytes + " MB."
```

Answer:

(i) Calculation:

1 GB = 1000 MB

Number of videos that could be stored = (1 GB / 100 MB) = 10

Therefore, William can store 10 videos on his computer if each video is 100MB in size.

(ii) Algorithm using pseudocode:

Ask the user to input the file size in megabytes.

Multiply the file size by 1,000,000 to convert it to bytes.

Print the result in a user-friendly format by dividing the bytes by 1,048,576 and rounding to two decimal places, and adding "MB" and "bytes" to the output.

Pseudocode:

Input file_size_in_mb

Set bytes = file_size_in_mb * 1000000

Set result = round((bytes / 1048576), 2)

Output "There are " + result + " MB " + bytes + " bytes".

Graph a line with a slope of 3/4 that contains the point (2,-3)

Answers

Step-by-step explanation:

[tex] - 3 = \frac{3}{4} (2) + b[/tex]

[tex] - \frac{6}{2} = \frac{3}{2} + b[/tex]

[tex]b = - \frac{9}{2} [/tex]

[tex]y = \frac{3}{4} x - \frac{9}{2} [/tex]

Alternately, starting at (2, -3), go up 3 units, then right 4 units, to (6, 0). Draw a line that goes through these points.

A sign on the roadway at the top of a mountain indicates that for the next 4 miles the grade is 9.5° (see figure). Find the change in elevation for a car descending the 4-mile stretch. (Round your answer to two decimal places.)

Answers

Step-by-step explanation:

We can use trigonometry to solve this problem. The grade of the road is given as an angle of 9.5°. This means that for every 1 unit of horizontal distance traveled, the elevation changes by the tangent of 9.5°.

We want to find the change in elevation for a car descending the 4-mile stretch. To do this, we can multiply the length of the road by the tangent of the grade angle:

Change in elevation = 4 miles x tan(9.5°)

Using a calculator, we can find that tan(9.5°) = 0.1664. Substituting this value into the equation, we get:

Change in elevation = 4 miles x 0.1664 ≈ 0.67 milesTo convert this to feet, we can multiply by the number of feet in a mile:

Change in elevation = 0.67 miles x 5,280 feet/mile ≈ 3,539 feetTherefore, the change in elevation for a car descending the 4-mile stretch is approximately 3,539 feet.

Jenny received a $1900 bonus. She decided to invest it in a 5-year certificate of deposit (CD) with an annual interest rate of 1.47% compounded quarterly.
Answer the questions below. Do not round any intermediate computations, and round your final answers to the nearest cent. If necessary, refer to the
list of financial formulas.
(a) Assuming no withdrawals are made, how much money is in Jenny's account
after 5 years?
$
(b) How much interest is earned on Jenny's investment after 5 years?
$

Answers

After Jenny invested the $1900 in a 5-year CD,

The money Jenny will have in her account after 5-years is $2400The interest she earned after 5-years is $500

A) Given principal money (P) = $1900

Rate of interest (r) = 1.47% / 100 = 0.047

number of times interest compounded per year (n) = 4

number of years (t) = 5

Amount Jenny will have after 5-years (A) = [tex]p(1+r/n)^{nt}[/tex]

A = [tex]1900(1+0.047/4)^{4*5}[/tex]

A = [tex]1900(1 + 0.01175)^{20}[/tex]

A = [tex]1900(1.01175)^{20}[/tex]

A = 1900(1.2631) ≅ $2400

B) Interest earned after 5 years = $2400 - $1900 = $500.

To know more about interest problems,

https://brainly.com/question/25720319

#SPJ1

A pitcher contains 3 quarts of iced
tea. John is going to pour the iced tea
into glasses that each hold 1½ cups.
How many glasses will John fill?

Answers

John will be able to fill 8 glasses with iced tea.

How many cups are in 3 quarts of iced tea?

To convert quarts to cups, we need to multiply the number of quarts by 4, since 1 quart is equal to 4 cups. So, 3 quarts of iced tea is equivalent to 3 x 4 = 12 cups of iced tea.

Since each glass holds 1½ cups of iced tea, John will need to divide the total cups of iced tea by the capacity of each glass: 12 cups / 1.5 cups/glass = 8 glasses.

Therefore, John will fill 8 glasses with iced tea.

Read more about iced tea

brainly.com/question/30179276

#SPJ1

Here is your sketch of a cylinder with a radius of 3 and
a height of 10.
Calculate the volume of the cylinder.
Express your answer in terms of it.

Answers

The volume of the cylinder is 90π cubic units.

To calculate the volume of a cylinder with a radius of 3 and a height of 10, you can use the following formula:

Volume = π × r² × h

Where:

- Volume represents the volume of the cylinder,

- π (pi) is a mathematical constant approximately equal to 3.14159,

- r is the radius of the cylinder (in this case, 3), and

- h is the height of the cylinder (in this case, 10).

Now, plug in the given values:

Volume = π × (3)² × 10

Simplify the expression:

Volume = π × 9 × 10

Volume = 90π

So, the volume of the cylinder is 90π cubic units.

Learn more about volume here,

https://brainly.com/question/218706

#SPJ1

URGENT PLEASE ANSWER!!! I INCLUDED THE GRAPH!
Graph the line y = 4/3x + 1.
Use the line tool and select two points on the line.

Answers

We have (0, 1) and (3, 5) as two points on the line y = 4/3x + 1.

Selecting two points on the line.

The equation y = 4/3x + 1 represents a line in slope-intercept form, where the coefficient of x (4/3) represents the slope of the line and the constant term (1) represents the y-intercept.

To select two points on the line, we can choose any values for x and use the equation to find the corresponding values of y.

For example, let's choose x = 0:

y = 4/3x + 1

y = 4/3(0) + 1

y = 1

So one point on the line is (0, 1).

Now let's choose x = 3:

y = 4/3x + 1

y = 4/3(3) + 1

y = 5

So another point on the line is (3, 5).

Since both equations hold true, we can conclude that (0, 1) and (3, 5) are two points on the line y = 4/3x + 1.

Read more about linear relation at

https://brainly.com/question/30318449

#SPJ1

Other Questions
Q5) h) Distinguish between octets and bytes. Given f(x) =-3x^2 - 2r Find f(8) find a. round to the nearest tenth. a=? cm In a bubble sort, on each pass through the list that must be sorted, you can stop making pair comparisons _____. A. one comparison later B. one comparison sooner C. two comparison sooner D. two comparison later What is the length of the diagonal from P to Q? A summary of two stocks is shown.52W high 52W low Name of Stock Symbol High Low Close37.18 29.39 Zycodec ZYO 39.06 32.73 34.9511.76 7.89 Unix Co UNX 16.12 12.11 15.78Last year, a stockholder purchased 25 shares of Zycodec at its lowest price of the year. What is the gain or loss if the stock is sold at the current closing price? The gain is $55.75. The loss is $55.75. The gain is $139.00. The loss is $139.00. If Jane eats her dinner, then the family goes to the beach.The family goes to the beach.Therefore, Jane eats her dinner. Assume the following data: Risk-free rate = 4.0 percent; average risk premium = 7.7 percent. Calculate the required rate of return for the risky asset. Applying Basic Probability RulesThe probability model shows the distribution of flavors of a chewable multivitamin.CherryStrawberry0.270.25FlavorLemonOrangeProbability0.28Suppose you select a multivitamin from the bottle without looking. Find each probability. Enter your answers asdecimals rounded to the hundredths place.0.20 Two similar solids have edges of 4 feet in 24 feet if the smaller saw that has a volume of 16 ft. find the volume of the other solid? A traditional light bulb gives out heat and light. It makes 5, J,5J of light for every 60, J,60J of electricity it uses. How much heat does it make? Describe the carbocation in terms of hybridization, structure and orbital characteristics. The political and military alliance imposed on Eastern Europe by the Soviet Union in the aftermath of World War II was known as A number k multiplied by 4 is less than -3/2 Solving systems by eliminations; finding the coeficientsplease write all the problems down on paper, 10 points for each problem, and Brainliest What is a primary distinction between stimulus generalization involving common-element stimulus classes and stimulus generalization involving stimulus equivalence classes? Which decision is an example of a consumer facing switching costs?A. Vijay must decide between Joe's soy ketchup vs. HJ Heinz tomato ketchupB. Vijay goes without a condimentC. Vijay decides he wants a wrap insteadD. Vijay must decide between ketchup, mustard, relish or other condiments Song: Fishtail by Lana Del ReyWhy do you think the artist named the song what they did?What is the attitude of the speaker/singer?Identify important tools and turnsWhat do you think the main theme or message of the song is? (c) Evaluate Are Spiegelman's techniques effective? Why or why not? The husband of a client with cervical cancer says to the nurse, "The doctor told my wife that her cancer is curable. Is he just trying to make us feel better?" Which would be the nurse's most accurate response?A. "When cervical cancer is detected early and treated aggressively, the cure rate is almost 100%"B. "The 5-year survival rate is about 75%, which makes the odds pretty good."C. "Saying a cancer is curable means that 50% of all women with the cancer survive at least 5 years."D. "Cancers of the female reproductive tract tend to be slow-growing and respond well to treatment."